Criterion for weakly lower semicontinuity in problems of calculus of variations












2












$begingroup$


Let’s say I have to minimize a certain integral functional $F(u)= int_{Omega} L(x,u,nabla u)$ with $Omega$ regular and $u$ in a certain Sobolev space (don’t really care to be precise, I just want a general idea). To prove there exists a minimum, let’s say I want to use the direct method. No problems for compactness, but when I want the lower semicontinuity for weakly convergent sequences (the derivatives, typically) I am kinda lost. I know the norm is weakly lower semicontinuous, but is there a simple general criterion to have weakly lower semicontinuity, in general?










share|cite|improve this question









$endgroup$












  • $begingroup$
    If I'm remembering correctly a convexity assumption on the lagrangian $L$ is sufficient to get the w.l.s.c. I think it's in Evans's book? Or maybe Jost's.
    $endgroup$
    – Jose27
    Dec 31 '18 at 19:08










  • $begingroup$
    Convexity and strong semicontinuity is enough, indeed. The problem is that in dimensions greater than 1 you usually only have $L^p$ convergence for the sequence. I mean, if I had uniform convergence then a a continuous Lagrangian would work. But with $L^p$ convergence, how do I have strong lower semicontinuity given strong $L^p$ convergence?
    $endgroup$
    – tommy1996q
    Dec 31 '18 at 19:34
















2












$begingroup$


Let’s say I have to minimize a certain integral functional $F(u)= int_{Omega} L(x,u,nabla u)$ with $Omega$ regular and $u$ in a certain Sobolev space (don’t really care to be precise, I just want a general idea). To prove there exists a minimum, let’s say I want to use the direct method. No problems for compactness, but when I want the lower semicontinuity for weakly convergent sequences (the derivatives, typically) I am kinda lost. I know the norm is weakly lower semicontinuous, but is there a simple general criterion to have weakly lower semicontinuity, in general?










share|cite|improve this question









$endgroup$












  • $begingroup$
    If I'm remembering correctly a convexity assumption on the lagrangian $L$ is sufficient to get the w.l.s.c. I think it's in Evans's book? Or maybe Jost's.
    $endgroup$
    – Jose27
    Dec 31 '18 at 19:08










  • $begingroup$
    Convexity and strong semicontinuity is enough, indeed. The problem is that in dimensions greater than 1 you usually only have $L^p$ convergence for the sequence. I mean, if I had uniform convergence then a a continuous Lagrangian would work. But with $L^p$ convergence, how do I have strong lower semicontinuity given strong $L^p$ convergence?
    $endgroup$
    – tommy1996q
    Dec 31 '18 at 19:34














2












2








2


0



$begingroup$


Let’s say I have to minimize a certain integral functional $F(u)= int_{Omega} L(x,u,nabla u)$ with $Omega$ regular and $u$ in a certain Sobolev space (don’t really care to be precise, I just want a general idea). To prove there exists a minimum, let’s say I want to use the direct method. No problems for compactness, but when I want the lower semicontinuity for weakly convergent sequences (the derivatives, typically) I am kinda lost. I know the norm is weakly lower semicontinuous, but is there a simple general criterion to have weakly lower semicontinuity, in general?










share|cite|improve this question









$endgroup$




Let’s say I have to minimize a certain integral functional $F(u)= int_{Omega} L(x,u,nabla u)$ with $Omega$ regular and $u$ in a certain Sobolev space (don’t really care to be precise, I just want a general idea). To prove there exists a minimum, let’s say I want to use the direct method. No problems for compactness, but when I want the lower semicontinuity for weakly convergent sequences (the derivatives, typically) I am kinda lost. I know the norm is weakly lower semicontinuous, but is there a simple general criterion to have weakly lower semicontinuity, in general?







optimization sobolev-spaces calculus-of-variations






share|cite|improve this question













share|cite|improve this question











share|cite|improve this question




share|cite|improve this question










asked Dec 31 '18 at 15:29









tommy1996qtommy1996q

604415




604415












  • $begingroup$
    If I'm remembering correctly a convexity assumption on the lagrangian $L$ is sufficient to get the w.l.s.c. I think it's in Evans's book? Or maybe Jost's.
    $endgroup$
    – Jose27
    Dec 31 '18 at 19:08










  • $begingroup$
    Convexity and strong semicontinuity is enough, indeed. The problem is that in dimensions greater than 1 you usually only have $L^p$ convergence for the sequence. I mean, if I had uniform convergence then a a continuous Lagrangian would work. But with $L^p$ convergence, how do I have strong lower semicontinuity given strong $L^p$ convergence?
    $endgroup$
    – tommy1996q
    Dec 31 '18 at 19:34


















  • $begingroup$
    If I'm remembering correctly a convexity assumption on the lagrangian $L$ is sufficient to get the w.l.s.c. I think it's in Evans's book? Or maybe Jost's.
    $endgroup$
    – Jose27
    Dec 31 '18 at 19:08










  • $begingroup$
    Convexity and strong semicontinuity is enough, indeed. The problem is that in dimensions greater than 1 you usually only have $L^p$ convergence for the sequence. I mean, if I had uniform convergence then a a continuous Lagrangian would work. But with $L^p$ convergence, how do I have strong lower semicontinuity given strong $L^p$ convergence?
    $endgroup$
    – tommy1996q
    Dec 31 '18 at 19:34
















$begingroup$
If I'm remembering correctly a convexity assumption on the lagrangian $L$ is sufficient to get the w.l.s.c. I think it's in Evans's book? Or maybe Jost's.
$endgroup$
– Jose27
Dec 31 '18 at 19:08




$begingroup$
If I'm remembering correctly a convexity assumption on the lagrangian $L$ is sufficient to get the w.l.s.c. I think it's in Evans's book? Or maybe Jost's.
$endgroup$
– Jose27
Dec 31 '18 at 19:08












$begingroup$
Convexity and strong semicontinuity is enough, indeed. The problem is that in dimensions greater than 1 you usually only have $L^p$ convergence for the sequence. I mean, if I had uniform convergence then a a continuous Lagrangian would work. But with $L^p$ convergence, how do I have strong lower semicontinuity given strong $L^p$ convergence?
$endgroup$
– tommy1996q
Dec 31 '18 at 19:34




$begingroup$
Convexity and strong semicontinuity is enough, indeed. The problem is that in dimensions greater than 1 you usually only have $L^p$ convergence for the sequence. I mean, if I had uniform convergence then a a continuous Lagrangian would work. But with $L^p$ convergence, how do I have strong lower semicontinuity given strong $L^p$ convergence?
$endgroup$
– tommy1996q
Dec 31 '18 at 19:34










0






active

oldest

votes











Your Answer





StackExchange.ifUsing("editor", function () {
return StackExchange.using("mathjaxEditing", function () {
StackExchange.MarkdownEditor.creationCallbacks.add(function (editor, postfix) {
StackExchange.mathjaxEditing.prepareWmdForMathJax(editor, postfix, [["$", "$"], ["\\(","\\)"]]);
});
});
}, "mathjax-editing");

StackExchange.ready(function() {
var channelOptions = {
tags: "".split(" "),
id: "69"
};
initTagRenderer("".split(" "), "".split(" "), channelOptions);

StackExchange.using("externalEditor", function() {
// Have to fire editor after snippets, if snippets enabled
if (StackExchange.settings.snippets.snippetsEnabled) {
StackExchange.using("snippets", function() {
createEditor();
});
}
else {
createEditor();
}
});

function createEditor() {
StackExchange.prepareEditor({
heartbeatType: 'answer',
autoActivateHeartbeat: false,
convertImagesToLinks: true,
noModals: true,
showLowRepImageUploadWarning: true,
reputationToPostImages: 10,
bindNavPrevention: true,
postfix: "",
imageUploader: {
brandingHtml: "Powered by u003ca class="icon-imgur-white" href="https://imgur.com/"u003eu003c/au003e",
contentPolicyHtml: "User contributions licensed under u003ca href="https://creativecommons.org/licenses/by-sa/3.0/"u003ecc by-sa 3.0 with attribution requiredu003c/au003e u003ca href="https://stackoverflow.com/legal/content-policy"u003e(content policy)u003c/au003e",
allowUrls: true
},
noCode: true, onDemand: true,
discardSelector: ".discard-answer"
,immediatelyShowMarkdownHelp:true
});


}
});














draft saved

draft discarded


















StackExchange.ready(
function () {
StackExchange.openid.initPostLogin('.new-post-login', 'https%3a%2f%2fmath.stackexchange.com%2fquestions%2f3057808%2fcriterion-for-weakly-lower-semicontinuity-in-problems-of-calculus-of-variations%23new-answer', 'question_page');
}
);

Post as a guest















Required, but never shown

























0






active

oldest

votes








0






active

oldest

votes









active

oldest

votes






active

oldest

votes
















draft saved

draft discarded




















































Thanks for contributing an answer to Mathematics Stack Exchange!


  • Please be sure to answer the question. Provide details and share your research!

But avoid



  • Asking for help, clarification, or responding to other answers.

  • Making statements based on opinion; back them up with references or personal experience.


Use MathJax to format equations. MathJax reference.


To learn more, see our tips on writing great answers.




draft saved


draft discarded














StackExchange.ready(
function () {
StackExchange.openid.initPostLogin('.new-post-login', 'https%3a%2f%2fmath.stackexchange.com%2fquestions%2f3057808%2fcriterion-for-weakly-lower-semicontinuity-in-problems-of-calculus-of-variations%23new-answer', 'question_page');
}
);

Post as a guest















Required, but never shown





















































Required, but never shown














Required, but never shown












Required, but never shown







Required, but never shown

































Required, but never shown














Required, but never shown












Required, but never shown







Required, but never shown







Popular posts from this blog

How do I know what Microsoft account the skydrive app is syncing to?

When does type information flow backwards in C++?

Grease: Live!